How to show $(exists x)( forall y)varphirightarrow( forall y)(exists x)varphi $ is logically valid











up vote
6
down vote

favorite
1













How to show $(exists x)( forall y)varphirightarrow( forall y)(exists x)varphi $ is logically valid




Here is my attempt:



Assume it's not logically valid. Then, there's an interpretation $mathscr{M}$ for which it's not true. Hence, there's a sequence $vec a$ in the domain $M$ of $mathscr{M}$ such that 1) $ vec a$ satisfies $(exists x)( forall y)varphi$ and 2) $vec a$ doesn't satisfy $( forall y)(exists x)varphi$



1) $vec a$ satisfies $(exists x)( forall y)varphi$ $iff$ $vec a$ doesn't satisfy $(forall x)( exists y)negvarphi$ $iff$ $vec a$ doesn't satisfy $( exists y)negvarphi$ $iff$ $vec a$ satisfies $( forall y)varphi$ $iff$ $vec a$ satisfies $varphi$



2) $vec a$ doesn't satisfy $( forall y)(exists x)varphi$ $iff$ $vec a$ doesn't satisfy $(exists x)varphi$ $iff$ $vec a$ satisfies $(forall x)negvarphi$ $iff$ $vec a$ satisfies $negvarphi$



Since $vec a$ satisfies both $varphi$ and $negvarphi$, which is a contradiction, then the formula must be valid.



Is it correct to show it this way?




Edit:
1) If $vec a$ satisfies $(exists x)( forall y)varphi$, i.e $neg forall x(negforall y(varphi))$, then we have $vec a$ does not satisfy $forall x(negforall y(varphi))$ . Then, there is at least one sequence $vec a'$ differing from $vec a$ in at most the ith component not satisfying $negforall y(varphi)$. Then, that means $vec a'$ satisfies $( forall y)varphi$.
2) If $vec a$ doesn't satisfy $( forall y)(exists x)varphi$, i.e $( forall y)neg(forall x)(negvarphi)$, then there is at least one sequence $vec a''$ differing from $vec a$ in at most the jth component not satisfying $neg(forall x)(negvarphi)$. Then, that means $vec a''$ satisfies $(forall x)negvarphi$.

Now, we have $vec a'$ satisfies $( forall y)varphi$ and $vec a''$ satisfies $(forall x)negvarphi$. Then, there is at least one sequence $vec a'''$ differing from $vec a$ in at most the $i$th and $j$th component satisfying $varphi$ and satisfying $negvarphi$, which is a contradiction, then the formula must be valid.




This is from the book, so I applied what's written in $2$ actually
enter image description here










share|cite|improve this question
























  • You can see Example 1.9.3, page 37 of Christopher Leary & Lars Kristiansen, A Friendly Introduction to Mathematical Logic (2nd ed 2015).
    – Mauro ALLEGRANZA
    Nov 23 at 20:43










  • It's very nice, but it uses a bit different convention from the one we use. Does mine still count, though? :)
    – Leyla Alkan
    Nov 23 at 20:49












  • @LeylaAlkan Perhaps I'm just confused by your conventions (I assume $vec a$ is a global variable assignment?), but what you've written does not seem to make sense. For instance, $vec a$ satisfying $exists x forall y varphi$ does not imply $vec a$ satisfies $phi.$ It holds for some $x$ (and any $y$) so you need to change the variable assignment to assign the appropriate value to $x.$
    – spaceisdarkgreen
    Nov 23 at 21:15












  • See my edit where I explain why I did it this way @spaceisdarkgreen
    – Leyla Alkan
    Nov 23 at 21:19










  • @LeylaAlkan That's Mendelson, I assume? I don't see any resemblance between what's written in $2$ and what you wrote. (You're going to have to use the rule for quantifiers at some point...)
    – spaceisdarkgreen
    Nov 23 at 21:20

















up vote
6
down vote

favorite
1













How to show $(exists x)( forall y)varphirightarrow( forall y)(exists x)varphi $ is logically valid




Here is my attempt:



Assume it's not logically valid. Then, there's an interpretation $mathscr{M}$ for which it's not true. Hence, there's a sequence $vec a$ in the domain $M$ of $mathscr{M}$ such that 1) $ vec a$ satisfies $(exists x)( forall y)varphi$ and 2) $vec a$ doesn't satisfy $( forall y)(exists x)varphi$



1) $vec a$ satisfies $(exists x)( forall y)varphi$ $iff$ $vec a$ doesn't satisfy $(forall x)( exists y)negvarphi$ $iff$ $vec a$ doesn't satisfy $( exists y)negvarphi$ $iff$ $vec a$ satisfies $( forall y)varphi$ $iff$ $vec a$ satisfies $varphi$



2) $vec a$ doesn't satisfy $( forall y)(exists x)varphi$ $iff$ $vec a$ doesn't satisfy $(exists x)varphi$ $iff$ $vec a$ satisfies $(forall x)negvarphi$ $iff$ $vec a$ satisfies $negvarphi$



Since $vec a$ satisfies both $varphi$ and $negvarphi$, which is a contradiction, then the formula must be valid.



Is it correct to show it this way?




Edit:
1) If $vec a$ satisfies $(exists x)( forall y)varphi$, i.e $neg forall x(negforall y(varphi))$, then we have $vec a$ does not satisfy $forall x(negforall y(varphi))$ . Then, there is at least one sequence $vec a'$ differing from $vec a$ in at most the ith component not satisfying $negforall y(varphi)$. Then, that means $vec a'$ satisfies $( forall y)varphi$.
2) If $vec a$ doesn't satisfy $( forall y)(exists x)varphi$, i.e $( forall y)neg(forall x)(negvarphi)$, then there is at least one sequence $vec a''$ differing from $vec a$ in at most the jth component not satisfying $neg(forall x)(negvarphi)$. Then, that means $vec a''$ satisfies $(forall x)negvarphi$.

Now, we have $vec a'$ satisfies $( forall y)varphi$ and $vec a''$ satisfies $(forall x)negvarphi$. Then, there is at least one sequence $vec a'''$ differing from $vec a$ in at most the $i$th and $j$th component satisfying $varphi$ and satisfying $negvarphi$, which is a contradiction, then the formula must be valid.




This is from the book, so I applied what's written in $2$ actually
enter image description here










share|cite|improve this question
























  • You can see Example 1.9.3, page 37 of Christopher Leary & Lars Kristiansen, A Friendly Introduction to Mathematical Logic (2nd ed 2015).
    – Mauro ALLEGRANZA
    Nov 23 at 20:43










  • It's very nice, but it uses a bit different convention from the one we use. Does mine still count, though? :)
    – Leyla Alkan
    Nov 23 at 20:49












  • @LeylaAlkan Perhaps I'm just confused by your conventions (I assume $vec a$ is a global variable assignment?), but what you've written does not seem to make sense. For instance, $vec a$ satisfying $exists x forall y varphi$ does not imply $vec a$ satisfies $phi.$ It holds for some $x$ (and any $y$) so you need to change the variable assignment to assign the appropriate value to $x.$
    – spaceisdarkgreen
    Nov 23 at 21:15












  • See my edit where I explain why I did it this way @spaceisdarkgreen
    – Leyla Alkan
    Nov 23 at 21:19










  • @LeylaAlkan That's Mendelson, I assume? I don't see any resemblance between what's written in $2$ and what you wrote. (You're going to have to use the rule for quantifiers at some point...)
    – spaceisdarkgreen
    Nov 23 at 21:20















up vote
6
down vote

favorite
1









up vote
6
down vote

favorite
1






1






How to show $(exists x)( forall y)varphirightarrow( forall y)(exists x)varphi $ is logically valid




Here is my attempt:



Assume it's not logically valid. Then, there's an interpretation $mathscr{M}$ for which it's not true. Hence, there's a sequence $vec a$ in the domain $M$ of $mathscr{M}$ such that 1) $ vec a$ satisfies $(exists x)( forall y)varphi$ and 2) $vec a$ doesn't satisfy $( forall y)(exists x)varphi$



1) $vec a$ satisfies $(exists x)( forall y)varphi$ $iff$ $vec a$ doesn't satisfy $(forall x)( exists y)negvarphi$ $iff$ $vec a$ doesn't satisfy $( exists y)negvarphi$ $iff$ $vec a$ satisfies $( forall y)varphi$ $iff$ $vec a$ satisfies $varphi$



2) $vec a$ doesn't satisfy $( forall y)(exists x)varphi$ $iff$ $vec a$ doesn't satisfy $(exists x)varphi$ $iff$ $vec a$ satisfies $(forall x)negvarphi$ $iff$ $vec a$ satisfies $negvarphi$



Since $vec a$ satisfies both $varphi$ and $negvarphi$, which is a contradiction, then the formula must be valid.



Is it correct to show it this way?




Edit:
1) If $vec a$ satisfies $(exists x)( forall y)varphi$, i.e $neg forall x(negforall y(varphi))$, then we have $vec a$ does not satisfy $forall x(negforall y(varphi))$ . Then, there is at least one sequence $vec a'$ differing from $vec a$ in at most the ith component not satisfying $negforall y(varphi)$. Then, that means $vec a'$ satisfies $( forall y)varphi$.
2) If $vec a$ doesn't satisfy $( forall y)(exists x)varphi$, i.e $( forall y)neg(forall x)(negvarphi)$, then there is at least one sequence $vec a''$ differing from $vec a$ in at most the jth component not satisfying $neg(forall x)(negvarphi)$. Then, that means $vec a''$ satisfies $(forall x)negvarphi$.

Now, we have $vec a'$ satisfies $( forall y)varphi$ and $vec a''$ satisfies $(forall x)negvarphi$. Then, there is at least one sequence $vec a'''$ differing from $vec a$ in at most the $i$th and $j$th component satisfying $varphi$ and satisfying $negvarphi$, which is a contradiction, then the formula must be valid.




This is from the book, so I applied what's written in $2$ actually
enter image description here










share|cite|improve this question
















How to show $(exists x)( forall y)varphirightarrow( forall y)(exists x)varphi $ is logically valid




Here is my attempt:



Assume it's not logically valid. Then, there's an interpretation $mathscr{M}$ for which it's not true. Hence, there's a sequence $vec a$ in the domain $M$ of $mathscr{M}$ such that 1) $ vec a$ satisfies $(exists x)( forall y)varphi$ and 2) $vec a$ doesn't satisfy $( forall y)(exists x)varphi$



1) $vec a$ satisfies $(exists x)( forall y)varphi$ $iff$ $vec a$ doesn't satisfy $(forall x)( exists y)negvarphi$ $iff$ $vec a$ doesn't satisfy $( exists y)negvarphi$ $iff$ $vec a$ satisfies $( forall y)varphi$ $iff$ $vec a$ satisfies $varphi$



2) $vec a$ doesn't satisfy $( forall y)(exists x)varphi$ $iff$ $vec a$ doesn't satisfy $(exists x)varphi$ $iff$ $vec a$ satisfies $(forall x)negvarphi$ $iff$ $vec a$ satisfies $negvarphi$



Since $vec a$ satisfies both $varphi$ and $negvarphi$, which is a contradiction, then the formula must be valid.



Is it correct to show it this way?




Edit:
1) If $vec a$ satisfies $(exists x)( forall y)varphi$, i.e $neg forall x(negforall y(varphi))$, then we have $vec a$ does not satisfy $forall x(negforall y(varphi))$ . Then, there is at least one sequence $vec a'$ differing from $vec a$ in at most the ith component not satisfying $negforall y(varphi)$. Then, that means $vec a'$ satisfies $( forall y)varphi$.
2) If $vec a$ doesn't satisfy $( forall y)(exists x)varphi$, i.e $( forall y)neg(forall x)(negvarphi)$, then there is at least one sequence $vec a''$ differing from $vec a$ in at most the jth component not satisfying $neg(forall x)(negvarphi)$. Then, that means $vec a''$ satisfies $(forall x)negvarphi$.

Now, we have $vec a'$ satisfies $( forall y)varphi$ and $vec a''$ satisfies $(forall x)negvarphi$. Then, there is at least one sequence $vec a'''$ differing from $vec a$ in at most the $i$th and $j$th component satisfying $varphi$ and satisfying $negvarphi$, which is a contradiction, then the formula must be valid.




This is from the book, so I applied what's written in $2$ actually
enter image description here







logic predicate-logic






share|cite|improve this question















share|cite|improve this question













share|cite|improve this question




share|cite|improve this question








edited Nov 25 at 20:56

























asked Nov 23 at 20:19









Leyla Alkan

1,5411723




1,5411723












  • You can see Example 1.9.3, page 37 of Christopher Leary & Lars Kristiansen, A Friendly Introduction to Mathematical Logic (2nd ed 2015).
    – Mauro ALLEGRANZA
    Nov 23 at 20:43










  • It's very nice, but it uses a bit different convention from the one we use. Does mine still count, though? :)
    – Leyla Alkan
    Nov 23 at 20:49












  • @LeylaAlkan Perhaps I'm just confused by your conventions (I assume $vec a$ is a global variable assignment?), but what you've written does not seem to make sense. For instance, $vec a$ satisfying $exists x forall y varphi$ does not imply $vec a$ satisfies $phi.$ It holds for some $x$ (and any $y$) so you need to change the variable assignment to assign the appropriate value to $x.$
    – spaceisdarkgreen
    Nov 23 at 21:15












  • See my edit where I explain why I did it this way @spaceisdarkgreen
    – Leyla Alkan
    Nov 23 at 21:19










  • @LeylaAlkan That's Mendelson, I assume? I don't see any resemblance between what's written in $2$ and what you wrote. (You're going to have to use the rule for quantifiers at some point...)
    – spaceisdarkgreen
    Nov 23 at 21:20




















  • You can see Example 1.9.3, page 37 of Christopher Leary & Lars Kristiansen, A Friendly Introduction to Mathematical Logic (2nd ed 2015).
    – Mauro ALLEGRANZA
    Nov 23 at 20:43










  • It's very nice, but it uses a bit different convention from the one we use. Does mine still count, though? :)
    – Leyla Alkan
    Nov 23 at 20:49












  • @LeylaAlkan Perhaps I'm just confused by your conventions (I assume $vec a$ is a global variable assignment?), but what you've written does not seem to make sense. For instance, $vec a$ satisfying $exists x forall y varphi$ does not imply $vec a$ satisfies $phi.$ It holds for some $x$ (and any $y$) so you need to change the variable assignment to assign the appropriate value to $x.$
    – spaceisdarkgreen
    Nov 23 at 21:15












  • See my edit where I explain why I did it this way @spaceisdarkgreen
    – Leyla Alkan
    Nov 23 at 21:19










  • @LeylaAlkan That's Mendelson, I assume? I don't see any resemblance between what's written in $2$ and what you wrote. (You're going to have to use the rule for quantifiers at some point...)
    – spaceisdarkgreen
    Nov 23 at 21:20


















You can see Example 1.9.3, page 37 of Christopher Leary & Lars Kristiansen, A Friendly Introduction to Mathematical Logic (2nd ed 2015).
– Mauro ALLEGRANZA
Nov 23 at 20:43




You can see Example 1.9.3, page 37 of Christopher Leary & Lars Kristiansen, A Friendly Introduction to Mathematical Logic (2nd ed 2015).
– Mauro ALLEGRANZA
Nov 23 at 20:43












It's very nice, but it uses a bit different convention from the one we use. Does mine still count, though? :)
– Leyla Alkan
Nov 23 at 20:49






It's very nice, but it uses a bit different convention from the one we use. Does mine still count, though? :)
– Leyla Alkan
Nov 23 at 20:49














@LeylaAlkan Perhaps I'm just confused by your conventions (I assume $vec a$ is a global variable assignment?), but what you've written does not seem to make sense. For instance, $vec a$ satisfying $exists x forall y varphi$ does not imply $vec a$ satisfies $phi.$ It holds for some $x$ (and any $y$) so you need to change the variable assignment to assign the appropriate value to $x.$
– spaceisdarkgreen
Nov 23 at 21:15






@LeylaAlkan Perhaps I'm just confused by your conventions (I assume $vec a$ is a global variable assignment?), but what you've written does not seem to make sense. For instance, $vec a$ satisfying $exists x forall y varphi$ does not imply $vec a$ satisfies $phi.$ It holds for some $x$ (and any $y$) so you need to change the variable assignment to assign the appropriate value to $x.$
– spaceisdarkgreen
Nov 23 at 21:15














See my edit where I explain why I did it this way @spaceisdarkgreen
– Leyla Alkan
Nov 23 at 21:19




See my edit where I explain why I did it this way @spaceisdarkgreen
– Leyla Alkan
Nov 23 at 21:19












@LeylaAlkan That's Mendelson, I assume? I don't see any resemblance between what's written in $2$ and what you wrote. (You're going to have to use the rule for quantifiers at some point...)
– spaceisdarkgreen
Nov 23 at 21:20






@LeylaAlkan That's Mendelson, I assume? I don't see any resemblance between what's written in $2$ and what you wrote. (You're going to have to use the rule for quantifiers at some point...)
– spaceisdarkgreen
Nov 23 at 21:20












1 Answer
1






active

oldest

votes

















up vote
2
down vote



accepted










You're forgetting to use from item 4 the part that says the sequence differs in at most the $i$th component.



For instance, say you have a sequence $a$ satisfying a formula $psi = exists xforall y(varphi)$. Mendelson uses only the universal quantifier, then, actually, the above rendering of $psi$ is only syntactic sugar for $neg forall x(negforall y(varphi))$.



For a sequence $a$ to satisfy $psi$, it means that, by item 2, $a$ does not satisfy $forall x(negforall y(varphi))$.



For $a$ to not satifsy $forall x(negforall y(varphi))$, by item 4, it means that there is at least one sequence $a'$ differing from $a$ in at most the $i$th component (in wich $i$ is the index of the variable $x$) not satisfying $negforall y(varphi)$.





As for your edit, what you can get is actually




  1. A sequence $a'$ differing from $a$ in at most the $i$th position ($i$ being the index of $x$) satisfying $forall y (varphi)$.

  2. A sequence $a''$ differing from $a$ in at most the $j$th position ($j$ being the index of $y$) satisfying $forall x(negvarphi)$.


for $x$ and $y$ can be different variables.



To obtain a contradiction from this you can use item 4 (from Mendelson) to obtain another sequence, say $a'''$, differing from $a$ in at most the $i$th and $j$th position that will work for both 1. and 2. above.






share|cite|improve this answer























  • But the equivalent of $psi = (exists x)(forall y)varphi$ is $neg (forall x)(exists y)(neg varphi)$
    – Leyla Alkan
    Nov 23 at 22:23










  • @NoahSchweber Can you please check my edit? I think I cannot use the same $vec a'$ , but don't know how else to get a contradiction
    – Leyla Alkan
    Nov 24 at 11:35










  • @LeylaAlkan , sorry, my bad. Changed the order of quantifier and negation. Fixed.
    – Tarc
    Nov 24 at 15:52










  • I changed my edit according to your way, can you again check it to see if it's okay now? @Tarc
    – Leyla Alkan
    Nov 25 at 14:57










  • Yes, I think that will work, @LeylaAlkan. Maybe you can add that $a'''$ can be obtained from $a'$ by assigning to its $j$th position the value $a''(j)$, or, equivalently, from $a''$ by assigning to its $i$th position the value $a'(i)$.
    – Tarc
    Nov 25 at 20:35











Your Answer





StackExchange.ifUsing("editor", function () {
return StackExchange.using("mathjaxEditing", function () {
StackExchange.MarkdownEditor.creationCallbacks.add(function (editor, postfix) {
StackExchange.mathjaxEditing.prepareWmdForMathJax(editor, postfix, [["$", "$"], ["\\(","\\)"]]);
});
});
}, "mathjax-editing");

StackExchange.ready(function() {
var channelOptions = {
tags: "".split(" "),
id: "69"
};
initTagRenderer("".split(" "), "".split(" "), channelOptions);

StackExchange.using("externalEditor", function() {
// Have to fire editor after snippets, if snippets enabled
if (StackExchange.settings.snippets.snippetsEnabled) {
StackExchange.using("snippets", function() {
createEditor();
});
}
else {
createEditor();
}
});

function createEditor() {
StackExchange.prepareEditor({
heartbeatType: 'answer',
convertImagesToLinks: true,
noModals: true,
showLowRepImageUploadWarning: true,
reputationToPostImages: 10,
bindNavPrevention: true,
postfix: "",
imageUploader: {
brandingHtml: "Powered by u003ca class="icon-imgur-white" href="https://imgur.com/"u003eu003c/au003e",
contentPolicyHtml: "User contributions licensed under u003ca href="https://creativecommons.org/licenses/by-sa/3.0/"u003ecc by-sa 3.0 with attribution requiredu003c/au003e u003ca href="https://stackoverflow.com/legal/content-policy"u003e(content policy)u003c/au003e",
allowUrls: true
},
noCode: true, onDemand: true,
discardSelector: ".discard-answer"
,immediatelyShowMarkdownHelp:true
});


}
});














draft saved

draft discarded


















StackExchange.ready(
function () {
StackExchange.openid.initPostLogin('.new-post-login', 'https%3a%2f%2fmath.stackexchange.com%2fquestions%2f3010790%2fhow-to-show-exists-x-forall-y-varphi-rightarrow-forall-y-exists-x-va%23new-answer', 'question_page');
}
);

Post as a guest















Required, but never shown

























1 Answer
1






active

oldest

votes








1 Answer
1






active

oldest

votes









active

oldest

votes






active

oldest

votes








up vote
2
down vote



accepted










You're forgetting to use from item 4 the part that says the sequence differs in at most the $i$th component.



For instance, say you have a sequence $a$ satisfying a formula $psi = exists xforall y(varphi)$. Mendelson uses only the universal quantifier, then, actually, the above rendering of $psi$ is only syntactic sugar for $neg forall x(negforall y(varphi))$.



For a sequence $a$ to satisfy $psi$, it means that, by item 2, $a$ does not satisfy $forall x(negforall y(varphi))$.



For $a$ to not satifsy $forall x(negforall y(varphi))$, by item 4, it means that there is at least one sequence $a'$ differing from $a$ in at most the $i$th component (in wich $i$ is the index of the variable $x$) not satisfying $negforall y(varphi)$.





As for your edit, what you can get is actually




  1. A sequence $a'$ differing from $a$ in at most the $i$th position ($i$ being the index of $x$) satisfying $forall y (varphi)$.

  2. A sequence $a''$ differing from $a$ in at most the $j$th position ($j$ being the index of $y$) satisfying $forall x(negvarphi)$.


for $x$ and $y$ can be different variables.



To obtain a contradiction from this you can use item 4 (from Mendelson) to obtain another sequence, say $a'''$, differing from $a$ in at most the $i$th and $j$th position that will work for both 1. and 2. above.






share|cite|improve this answer























  • But the equivalent of $psi = (exists x)(forall y)varphi$ is $neg (forall x)(exists y)(neg varphi)$
    – Leyla Alkan
    Nov 23 at 22:23










  • @NoahSchweber Can you please check my edit? I think I cannot use the same $vec a'$ , but don't know how else to get a contradiction
    – Leyla Alkan
    Nov 24 at 11:35










  • @LeylaAlkan , sorry, my bad. Changed the order of quantifier and negation. Fixed.
    – Tarc
    Nov 24 at 15:52










  • I changed my edit according to your way, can you again check it to see if it's okay now? @Tarc
    – Leyla Alkan
    Nov 25 at 14:57










  • Yes, I think that will work, @LeylaAlkan. Maybe you can add that $a'''$ can be obtained from $a'$ by assigning to its $j$th position the value $a''(j)$, or, equivalently, from $a''$ by assigning to its $i$th position the value $a'(i)$.
    – Tarc
    Nov 25 at 20:35















up vote
2
down vote



accepted










You're forgetting to use from item 4 the part that says the sequence differs in at most the $i$th component.



For instance, say you have a sequence $a$ satisfying a formula $psi = exists xforall y(varphi)$. Mendelson uses only the universal quantifier, then, actually, the above rendering of $psi$ is only syntactic sugar for $neg forall x(negforall y(varphi))$.



For a sequence $a$ to satisfy $psi$, it means that, by item 2, $a$ does not satisfy $forall x(negforall y(varphi))$.



For $a$ to not satifsy $forall x(negforall y(varphi))$, by item 4, it means that there is at least one sequence $a'$ differing from $a$ in at most the $i$th component (in wich $i$ is the index of the variable $x$) not satisfying $negforall y(varphi)$.





As for your edit, what you can get is actually




  1. A sequence $a'$ differing from $a$ in at most the $i$th position ($i$ being the index of $x$) satisfying $forall y (varphi)$.

  2. A sequence $a''$ differing from $a$ in at most the $j$th position ($j$ being the index of $y$) satisfying $forall x(negvarphi)$.


for $x$ and $y$ can be different variables.



To obtain a contradiction from this you can use item 4 (from Mendelson) to obtain another sequence, say $a'''$, differing from $a$ in at most the $i$th and $j$th position that will work for both 1. and 2. above.






share|cite|improve this answer























  • But the equivalent of $psi = (exists x)(forall y)varphi$ is $neg (forall x)(exists y)(neg varphi)$
    – Leyla Alkan
    Nov 23 at 22:23










  • @NoahSchweber Can you please check my edit? I think I cannot use the same $vec a'$ , but don't know how else to get a contradiction
    – Leyla Alkan
    Nov 24 at 11:35










  • @LeylaAlkan , sorry, my bad. Changed the order of quantifier and negation. Fixed.
    – Tarc
    Nov 24 at 15:52










  • I changed my edit according to your way, can you again check it to see if it's okay now? @Tarc
    – Leyla Alkan
    Nov 25 at 14:57










  • Yes, I think that will work, @LeylaAlkan. Maybe you can add that $a'''$ can be obtained from $a'$ by assigning to its $j$th position the value $a''(j)$, or, equivalently, from $a''$ by assigning to its $i$th position the value $a'(i)$.
    – Tarc
    Nov 25 at 20:35













up vote
2
down vote



accepted







up vote
2
down vote



accepted






You're forgetting to use from item 4 the part that says the sequence differs in at most the $i$th component.



For instance, say you have a sequence $a$ satisfying a formula $psi = exists xforall y(varphi)$. Mendelson uses only the universal quantifier, then, actually, the above rendering of $psi$ is only syntactic sugar for $neg forall x(negforall y(varphi))$.



For a sequence $a$ to satisfy $psi$, it means that, by item 2, $a$ does not satisfy $forall x(negforall y(varphi))$.



For $a$ to not satifsy $forall x(negforall y(varphi))$, by item 4, it means that there is at least one sequence $a'$ differing from $a$ in at most the $i$th component (in wich $i$ is the index of the variable $x$) not satisfying $negforall y(varphi)$.





As for your edit, what you can get is actually




  1. A sequence $a'$ differing from $a$ in at most the $i$th position ($i$ being the index of $x$) satisfying $forall y (varphi)$.

  2. A sequence $a''$ differing from $a$ in at most the $j$th position ($j$ being the index of $y$) satisfying $forall x(negvarphi)$.


for $x$ and $y$ can be different variables.



To obtain a contradiction from this you can use item 4 (from Mendelson) to obtain another sequence, say $a'''$, differing from $a$ in at most the $i$th and $j$th position that will work for both 1. and 2. above.






share|cite|improve this answer














You're forgetting to use from item 4 the part that says the sequence differs in at most the $i$th component.



For instance, say you have a sequence $a$ satisfying a formula $psi = exists xforall y(varphi)$. Mendelson uses only the universal quantifier, then, actually, the above rendering of $psi$ is only syntactic sugar for $neg forall x(negforall y(varphi))$.



For a sequence $a$ to satisfy $psi$, it means that, by item 2, $a$ does not satisfy $forall x(negforall y(varphi))$.



For $a$ to not satifsy $forall x(negforall y(varphi))$, by item 4, it means that there is at least one sequence $a'$ differing from $a$ in at most the $i$th component (in wich $i$ is the index of the variable $x$) not satisfying $negforall y(varphi)$.





As for your edit, what you can get is actually




  1. A sequence $a'$ differing from $a$ in at most the $i$th position ($i$ being the index of $x$) satisfying $forall y (varphi)$.

  2. A sequence $a''$ differing from $a$ in at most the $j$th position ($j$ being the index of $y$) satisfying $forall x(negvarphi)$.


for $x$ and $y$ can be different variables.



To obtain a contradiction from this you can use item 4 (from Mendelson) to obtain another sequence, say $a'''$, differing from $a$ in at most the $i$th and $j$th position that will work for both 1. and 2. above.







share|cite|improve this answer














share|cite|improve this answer



share|cite|improve this answer








edited Nov 24 at 17:17

























answered Nov 23 at 21:56









Tarc

339411




339411












  • But the equivalent of $psi = (exists x)(forall y)varphi$ is $neg (forall x)(exists y)(neg varphi)$
    – Leyla Alkan
    Nov 23 at 22:23










  • @NoahSchweber Can you please check my edit? I think I cannot use the same $vec a'$ , but don't know how else to get a contradiction
    – Leyla Alkan
    Nov 24 at 11:35










  • @LeylaAlkan , sorry, my bad. Changed the order of quantifier and negation. Fixed.
    – Tarc
    Nov 24 at 15:52










  • I changed my edit according to your way, can you again check it to see if it's okay now? @Tarc
    – Leyla Alkan
    Nov 25 at 14:57










  • Yes, I think that will work, @LeylaAlkan. Maybe you can add that $a'''$ can be obtained from $a'$ by assigning to its $j$th position the value $a''(j)$, or, equivalently, from $a''$ by assigning to its $i$th position the value $a'(i)$.
    – Tarc
    Nov 25 at 20:35


















  • But the equivalent of $psi = (exists x)(forall y)varphi$ is $neg (forall x)(exists y)(neg varphi)$
    – Leyla Alkan
    Nov 23 at 22:23










  • @NoahSchweber Can you please check my edit? I think I cannot use the same $vec a'$ , but don't know how else to get a contradiction
    – Leyla Alkan
    Nov 24 at 11:35










  • @LeylaAlkan , sorry, my bad. Changed the order of quantifier and negation. Fixed.
    – Tarc
    Nov 24 at 15:52










  • I changed my edit according to your way, can you again check it to see if it's okay now? @Tarc
    – Leyla Alkan
    Nov 25 at 14:57










  • Yes, I think that will work, @LeylaAlkan. Maybe you can add that $a'''$ can be obtained from $a'$ by assigning to its $j$th position the value $a''(j)$, or, equivalently, from $a''$ by assigning to its $i$th position the value $a'(i)$.
    – Tarc
    Nov 25 at 20:35
















But the equivalent of $psi = (exists x)(forall y)varphi$ is $neg (forall x)(exists y)(neg varphi)$
– Leyla Alkan
Nov 23 at 22:23




But the equivalent of $psi = (exists x)(forall y)varphi$ is $neg (forall x)(exists y)(neg varphi)$
– Leyla Alkan
Nov 23 at 22:23












@NoahSchweber Can you please check my edit? I think I cannot use the same $vec a'$ , but don't know how else to get a contradiction
– Leyla Alkan
Nov 24 at 11:35




@NoahSchweber Can you please check my edit? I think I cannot use the same $vec a'$ , but don't know how else to get a contradiction
– Leyla Alkan
Nov 24 at 11:35












@LeylaAlkan , sorry, my bad. Changed the order of quantifier and negation. Fixed.
– Tarc
Nov 24 at 15:52




@LeylaAlkan , sorry, my bad. Changed the order of quantifier and negation. Fixed.
– Tarc
Nov 24 at 15:52












I changed my edit according to your way, can you again check it to see if it's okay now? @Tarc
– Leyla Alkan
Nov 25 at 14:57




I changed my edit according to your way, can you again check it to see if it's okay now? @Tarc
– Leyla Alkan
Nov 25 at 14:57












Yes, I think that will work, @LeylaAlkan. Maybe you can add that $a'''$ can be obtained from $a'$ by assigning to its $j$th position the value $a''(j)$, or, equivalently, from $a''$ by assigning to its $i$th position the value $a'(i)$.
– Tarc
Nov 25 at 20:35




Yes, I think that will work, @LeylaAlkan. Maybe you can add that $a'''$ can be obtained from $a'$ by assigning to its $j$th position the value $a''(j)$, or, equivalently, from $a''$ by assigning to its $i$th position the value $a'(i)$.
– Tarc
Nov 25 at 20:35


















draft saved

draft discarded




















































Thanks for contributing an answer to Mathematics Stack Exchange!


  • Please be sure to answer the question. Provide details and share your research!

But avoid



  • Asking for help, clarification, or responding to other answers.

  • Making statements based on opinion; back them up with references or personal experience.


Use MathJax to format equations. MathJax reference.


To learn more, see our tips on writing great answers.





Some of your past answers have not been well-received, and you're in danger of being blocked from answering.


Please pay close attention to the following guidance:


  • Please be sure to answer the question. Provide details and share your research!

But avoid



  • Asking for help, clarification, or responding to other answers.

  • Making statements based on opinion; back them up with references or personal experience.


To learn more, see our tips on writing great answers.




draft saved


draft discarded














StackExchange.ready(
function () {
StackExchange.openid.initPostLogin('.new-post-login', 'https%3a%2f%2fmath.stackexchange.com%2fquestions%2f3010790%2fhow-to-show-exists-x-forall-y-varphi-rightarrow-forall-y-exists-x-va%23new-answer', 'question_page');
}
);

Post as a guest















Required, but never shown





















































Required, but never shown














Required, but never shown












Required, but never shown







Required, but never shown

































Required, but never shown














Required, but never shown












Required, but never shown







Required, but never shown







Popular posts from this blog

Le Mesnil-Réaume

Ida-Boy-Ed-Garten

web3.py web3.isConnected() returns false always